User Avatar
cwetcholowsky623
Joined
Apr 2025
Subscription
Free
PrepTests ·
PT142.S4.Q21
User Avatar
cwetcholowsky623
Saturday, Dec 30 2023

I did not like (A) as a right answer choice because we technically don't know whether Hanlon's claim is backed up by extraordinary evidence. Certainly the fact that Hanlon is a trusted member of the community does not rise to the standard, but is there further evidence that speaks in Hanlon's favor? We do not know; the argument never says that Hanlon's status as a community member is the ONLY backup that Hanlon has for their claims.

To resolve this concern, the best way seems to be to restrict the domain we are talking about. Certainly in the argument in the stimulus Hanlon's status is the only evidence produced in Hanlon's favor; if we focus on this argument alone (A) does seem to apply.

PrepTests ·
PT143.S3.Q13
User Avatar
cwetcholowsky623
Friday, Dec 29 2023

(C) and (D) both go after the administrator's premise 'We have TAs only because this allows them to fund their education,' but (C) challenges this premise more effectively. (D) has more ambiguity ('most' could also just be 51%, and funding tuition does not have to mean also funding all of their education). By contrast, (C) in and of itself directly challenges the premise without being open to interpretation or requiring additional assumptions.

The stipends in (D) are also somewhat unclear to me. Are they identical to the financial compensations that the TAs receive for teaching? Or are these stipends independent of these compensations / an additional form of funding? The fact that (D) raises so many interpretative issues strikes me as a red flag.

Takeaways: Re-read this multiple times, color code the argument structure, really be careful with the answer choices and weigh them carefully against another. Choose the one that has the least problems / is not open to interpretation or in need of additional assumptions.

PrepTests ·
PT143.S3.Q9
User Avatar
cwetcholowsky623
Friday, Dec 29 2023

Notice how the LSAT is announcing this not as a parallel flaw question but as a parallel reasoning one, even though the argument arguably contains a belief / fact confusion (having no indication of signs for life vs. the signs do not exist). So the LSAT must intend this argument to run as follows:

(P1) LIFE -> SIGNS

(P2) Not SIGNS

(MC) Therefore, not LIFE

(E) mirrors this:

(P1) PLAN -> (MOVEMENTS or TRANSFER)

(P2) Not (MOVEMENTS or TRANSFER)

(MC) Therefore, not PLAN

(A) does not mirror the stimulus:

(P1) Know: SPY -> TRAITOR

(P2) Don't know: GENERAL -> TRAITOR

(MC) Therefore, GENERAL -> Not SPY

(A) in fact brings in the belief / fact confusion that the stimulus suggests, but (A) seems invalid and confuses the scope of the negation; it applies to the knowledge claim and not the general. Since the stimulus is supposed to amount to a valid argument (parallel reasoning, not parallel flaw!), (A) can't be right.

Takeaways: Read carefully; formalize in writing if there is time to get a better sense of the structures of the arguments being considered.

PrepTests ·
PT143.S2.P3.Q20
User Avatar
cwetcholowsky623
Friday, Dec 29 2023

On (Q20): We want to make the following claims compatible with another: (1) 'It is not the case that a pathological liar's lying gives you a sufficient reason to lie to them.' (2) For any rational being, if they treat you in a certain way, they in doing so authorize you to treat them in the same way.

Are these claims incompatible in the first place? Arguably yes, due to 'reason:' Passage A is saying, having a pathological liar lie to you does not give you a reason to lie back at them, whereas the first paragraph of B is saying that such liars authorize you to lie back at them.

(B): Pathological liars are not rational beings and thus fall outside the scope of the argument.

(D): Rights and duties to lie to pathological liars are different.

Why is (B) better here than (D)? (D) arguably would be very good if it said 'Rights and duties to pathological liars are different from another, being lied to only gives you a right to lie back at them, and having a right to lie to somebody is not a sufficient reason to in fact use this right.' That is, for (D) to work, we would need to add further assumptions, which the answer choice itself does not provide.

By contrast, (B) is less open to interpretation / requires no additional assumptions, and just straight up tells us that claim (1) does not fall within the scope of the argument in the first paragraph of passage B, thus making them at a minimum not incompatible.

Ranking: B > D > A, C, E

My difficulty in answering this question comes from the use of 'sufficient reason,' which I took to mean 'sufficient condition.' On this reading, it is not even clear whether the passages in fact are incompatible; just because someone authorizes me to lie to them does not obviously provide me with a sufficient condition that necessarily triggers a certain consequence. Even on this reading, however, (D) would be more open to interpretation than (B) and require more additional assumptions; (B) in itself still would be better than (D).

Takeaways: To get these sorts of questions right, first figure out where exactly the tension is: What specifically is the point where the passages conflict? Here, it would be the question 'If a pathological liar lies to you, should you lie back to them?,' whereby passage A says 'No' and the first paragraph of passage B says 'Yes.' To resolve this apparent incompatibility, we thus have to find an answer choice that either makes passage A compatible with the idea that we should in fact lie back to the liar, or that makes the first paragraph of B compatible with the suggestion that we should not lie back to them. Answer choice (B) goes for this second option by removing the pathological liar from the scope of the argument.

User Avatar
cwetcholowsky623
Monday, Jan 29 2024

I'm in PA and would love to stay in the loop!

PrepTests ·
PT144.S2.Q24
User Avatar
cwetcholowsky623
Wednesday, Sep 27 2023

This argument seems too idiosyncratic (incoherent?) to formalize it. The crucial thing to notice are that there are some term shift ('sells well' and 'popular,' 'written to give pleasure' and 'gives pleasure,' 'impart the truth' and 'being partially untrue'), which the author simply seems to gloss over. Moreover, there is only one answer choice that directly addresses one of these term shifts, so process of elimination is most likely the way to go here and probably the most realistic option to get this question right.

User Avatar
cwetcholowsky623
Thursday, Jan 25 2024

Honestly, I would focus on improving your GPA for now. The LSAT will still be there for you afterwards, but once your GPA is established you can't change it any more.

PrepTests ·
PT142.S2.Q15
User Avatar
cwetcholowsky623
Friday, Sep 22 2023

(P1) Often, there is only one kind of bodily structure that will allow a given animal type to satisfy a given need.

(P2) Moreover, different animal types often have similar needs.

(IC) Therefore, [...].

(MC) Therefore, it is unsurprising that different animal types often have similar bodily structures.

(IC) is most plausibly interpreted as 'Therefore, different animal types will often address their needs in similar ways.'

(B) approximates this more closely than (C), which lacks the idea of similarity and is formulated in stronger language.

(B) > (C) > (A), (D), (E)

User Avatar

Thursday, Apr 20 2023

cwetcholowsky623

PTF97.S1.Q18 – Roseville Courthouse

PT F97.S1.Q18 – Roseville Courthouse

We are asked to identify the point at issue / disagreement between Mayor Tyler and Councillor Simon. Tyler suggested to build a new courthouse for the city of Roseville in 1982 for a price of 26 million dollars, but ‘now’ in 1992 the price of the courthouse is 30 million. Tyler uses these premises to infer that Roseville would have saved 4 million dollars if the courthouse had been built in 1982, as suggested. Tyler also mentions in passing that the existing courthouse has been overcrowded.

Simon responds by bringing in the topic of inflation: The 26 million dollars that the courthouse would have costed in 1982 are equivalent to 37 million in 1992 dollars. Simon takes this to show that Roseville actually saved money by not building the courthouse. Simon also mentions in passing that the courthouse, had it actually been built, would have been underutilized.

There thus are at least two disagreements in this exchange, one much more overt than the other: (1) Roseville was right not to build the courthouse in 1982: Tyler disagrees, Simon agrees. (2) Had the courthouse been built, it would have been put to good use: Tyler agrees, Simon disagrees. The answer choices are tricky in that four of them purport to get at this first disagreement while not actually resolving it. Only one answer choice, the correct one, gets at the second disagreement and actually resolves it:

(A) This gets at Roseville’s actions going forward, does not directly relate to either disagreement.

(B) This gets at the issue of inflation adjusted prices, does not directly relate to either disagreement.

(C) This gets at the extent of Tyler’s responsibility, does not directly relate to either disagreement.

(D) This does get at the second disagreement and points out one issue where Tyler and Simon disagree: Would a new courthouse actually have been needed / been put to good use? Tyler agrees, as Tyler proclaims the present courthouse overcrowded, i.e. insufficient to serve Roseville’s existing population spatially. Simon disagrees; states that a hypothetical larger courthouse would have remained underutilized. The disagreement is subtle, but definitely present.

(E) This confuses the issue of inflation adjustment with financial upkeep, purports to get at the first disagreement but actually misrepresents information from the passage, in an apparent attempt to confuse test takers who did not select one of the previous answers the first time around.

Takeaway: This is a tricky question in that there are two disagreements only one of which gets resolved. The question stem arguably hints at this by speaking of ‘A point of disagreement,’ rather than of ‘The point of disagreement;’ i.e. the question stem leaves open the possibility of multiple disagreements. Nevertheless, this question demands some reflection. Read stimulus and answer choices more than once to get at the nuance of the issues at play. Do process of elimination for the wrong answer choices. If necessary, flag the question the first time around and return to it at the end of the section.

PrepTests ·
PT153.S1.P3.Q16
User Avatar
cwetcholowsky623
Saturday, Dec 16 2023

For (Q16), I would anticipate the main point to be something along the lines of "Incentivized confessions can be seriously problematic because they can encourage people to give false testimony and because juries may not always detect this." (B) approximates this to a certain extent because it points towards the problematic consequences of incentivized confessions. (C) and (E) may be descriptively accurate but do not actually state the main issue that the author is trying to highlight.

PrepTests ·
PT154.S4.Q24
User Avatar
cwetcholowsky623
Wednesday, Nov 15 2023

Stimulus:

(P1) (TRAUMA and Not PTSD) cor. CORTISOL↑

(MC) Therefore, TRAUMA => CORTISOL↑

This argument contains a possible correlation / causation confusion. To weaken this, show that the causal connection behind the observed correlation could be different. Also notice that the conclusion overgeneralizes from the premise: The premise is only about traumatized people without PTSD, but the conclusion talks about traumatized people generally. This might be another way to weaken this argument.

(B) states: (TRAUMA and CORTISOL↑) => Not PTSD

This suggests that it might not be the trauma that is causing the increased cortisol levels. Instead, the increased cortisol causes the absence of PTSD in cases of trauma. So we are getting a case of reverse causation, which weakens the original inference from the stimulus. (B) presents us with a very clear alternative explanation of the observed correlation.

Takeaways: Look out for scope shifts between premises and conclusion, possible sampling biases, and alternative causal explanations of a given correlation.

User Avatar

Friday, Apr 14 2023

cwetcholowsky623

PT16.S3.Q10 - Michelangelo Restaurations

The reason why the restorers want to make an exception for De Volterra is that De Volterra removed ALL layers of paint in the sections of the paintings where he made his addition before he made these additional. This suggests: De Volterra removed not only those layers of paint that people other than Michelangelo had added but also parts of Michelangelo's works themselves. This in turn means: If De Volterra's additions were removed as well, Michelangelo's underlying painting would not be revealed. Instead, there would just be a blank piece of wall, as De Volterra already removed those sections of Michelangelo's work that were there originally.

Making an exception for De Volterra thus arguably makes sense, because even an addition made by a painter other than Michelangelo would seem better than just have a blank piece of wall in the middle of Michelangelo's painting.

PrepTests ·
PT132.S4.Q24
User Avatar
cwetcholowsky623
Saturday, Oct 14 2023

(P1) Studies of arthritis did not find a correlation between the weather features supposedly determining arthritis pain levels and these pain levels themselves.

(P2) Arthritis patients furthermore could not identify a consistent time span between the supposed weather features determining their arthritis pain level and the actual onset of the pain.

(MC) Therefore, [MSS: if there is in fact a causal connection between certain weather features and arthritis pain levels, this connection must be somehow different from what the arthritis patients described].

MSS question; checking answer choices against this anticipated answer:

(A) The weather affects different arthritis sufferers after different time spans. Maybe, but here we are presuming that the weather in fact causally determines arthritis pain levels. For all we know, this is not guaranteed.

(B) Beliefs are causally determining arthritis pain levels. This seems unsupported by the argument; why would we get at beliefs specifically as an alternative cause? Is there a correlation between a particular weather phenomenon and temporal length? We don't know.

(C) Arthritis pain sufferers are merely imagining the correlations that they claim to exist. This is worded very strongly, but it does get at the '...if there is in fact a connection...' part of the anticipated answer. Keep this around!

(D) Some arthritis patients are more strongly affected by the weather than others. Maybe, but here we are again presuming that the patients reports are correct. For all we know, this may not be so.

(E) Scientific investigation of arthritis pain is impossible. This seems false, we know from the stimulus that there were studies.

After going through all the answer choices, (C) thus seems best because it is better than the available alternatives. Ranking: (C) > (A), (D) > (B) > (E). The strong wording definitely makes this an uncomfortable right answer choice.

Notice how the stimulus provides a hint about (C) being the correct answer though by saying 'those arthritis patients who were convinced of the existence of such a correlation.' By specifically picking out this subset of arthritis patients in this particular way, the stimulus suggests that there may also be arthritis patients who are not convinced that such a correlation exists, which in turn leads to (C).

Takeaways: Really read all individual answers attentively, and compare the available choices against each other. Select the answer choice that has the least problems.

PrepTests ·
PT132.S4.Q5
User Avatar
cwetcholowsky623
Saturday, Oct 14 2023

Stimulus

(P1) SKIN CANCER RATES ↑ cor. SUNSCREEN USAGE ↑

(MC) Therefore, not (SUNSCREEN => SKIN CANCER ↓)

One way to weaken this argument would be to find an alternative explanation of how increased skin cancer rates are compatible with increased sunscreen usage. Perhaps the sunscreen is in fact effective, but the population using it somehow diverges from the population getting skin cancer? Alternatively, there could be some cause other than the sun that those affected by cancer are exposed to even though they use sunscreen.

The correct answer (B) goes for a version of the anticipated divergence, though this divergence is temporal rather than population-oriented. Here, we learn: Skin cancer takes time to come about in those who had sunburns. Even though the people NOW spending time in the sun tend to use sunscreen and thus are protected, the people spending time in the sun IN THE PAST did not do so, and thus they still keep getting cancer. We thus reconcile the increased skin cancer rates with the increased use of sunscreen in a way that does not entail that sunscreen must be ineffective; we found an alternative causal explanation of the observed correlation.

(E) says: People who use sunscreen MOST REGULARLY are also those that are (take themselves to be) most susceptible to skin cancer. But does this group also keep getting affected by cancer disproportionally? We don't know; all the stimulus tells us is that overall skin cancer rates are increasing. Moreover, this answer choice is just about people 'believing themselves to be' most susceptible, we don't in fact know whether this belief is correct.

If (E) had said: Those most susceptible to skin cancer have grown increasingly UNLIKELY to use sun screen, (E) arguably would have weakened the argument by weakening the observed correlation. Alternatively, (E) could have said that those most vulnerable to skin cancer have actually become really well protected since they started using sunscreen. Neither of these alternatives would actually have provided an alternative explanation though. We only would have weakened the argument by attacking a premise, which is a rarer weakening option in LSAT contexts.

User Avatar

Friday, Apr 14 2023

cwetcholowsky623

PT16.S3.Q12 - Retina Scanners

PT16.S3.12 – Retina Scanners

This argument deals with retina scanners, machines that scan the blood vessel patterns in people’s eyes and stores these patterns, such that the scanners can recognized previously scanned patterns. The author furthermore posits that no two eyes have identical blood pattern vessels in their retinas, which seems to suggest that any given person has at least two such patterns, one for the left eye and one for the right one. The author then infers the conclusion that “[a] retina scanner can therefore be used successfully to determine for any person whether it has ever scanned a retina of that person before.”

We are supposed to identify a necessary assumption for this argument, i.e. an assumption that must be true for the conclusion to follow from the premises. Under timed conditions, I chose (B), which posits that everyone’s left and right eyes have identical patterns. I took this to be necessary for the conclusion to follow, due to conclusion’s scope (the conclusion is about “for any PERSON who ever had a retina scanned,” not about “for any given RETINA that ever has been scanned”). However, (B) seems to be false, for at least two reasons: (1) (B) goes against the information we get in the stimulus, where we are explicitly told that no two retinas have identical patterns. (2) (B) does not seem necessary for the rest of the claim that the conclusion seeks to establish (“Retina scanners allow you to answer the question, has one of the this person’s retinas ever been scanned?”). To make (B) a necessary condition, the conclusion would have to say something like “Even if you only scanned one of this person’s two retinas beforehand but not the other, retina scanners allow you to determine whether either of this person’s retinas has ever been scanned before.” However, (B) is not necessary for the way the conclusion is actually stated; the conclusion never says that the evidence to consider for any given person is a scan of only one of their retinas, as opposed to two.

The right answer choice (A) avoids this mistake by blocking another possible objection: What if people’s retina patterns change over time? Wouldn’t this make it impossible to recognize past scans later on, contrary to the argument’s conclusion suggests? (A) blocks this possible objection by establishing: Even if people get e.g. eye sicknesses, the patterns in their retinas remain unchanged over time.

PrepTests ·
PT135.S1.Q25
User Avatar
cwetcholowsky623
Thursday, Sep 14 2023

Stimulus:

(P1) COFFEE HOUSE -> P. PLACE

(P2) RESTAURANTS -> P. PLACE

(P3) (P. PLACE and WELL-DESIGNED) -m-> ARTWORK

(P4) (P. PLACE and WELL-DESIGNED -> COMFORTABLE -> SP. INTERIOR)

(A) (RESTAURANT ans SP. INTERIOR) -> COMFORTABLE. Does not follow.

(B) (P. PLACE and ARTWORK) -> WELL_DESIGNED. Does not follow

(C) (COFFEE HOUSE and WELL-DESIGNED) -m-> ARTWORK. Illicitly transfers a 'most' statement from '(WELL-DESIGNED and P. PLACE)' to a sub-category of (WELL-DESIGNED and P. PLACE).

(D) (COFFEEHOUSE and WELL-DESIGNED) -> SP. INTERIOR. Follows from (P1) and (P4).

(E) (COFFEE HOUSE and SP. INTERIOR) -> (P.PLACE and WELL-DESIGNED). Does not follow.

Takeaways: Be careful with 'most' statements. Don't confuse 'A -m-> B -> C; therefore, A -m-> C' (valid!) with 'A -> B -m-> C; therefore A -m-> C' (invalid!).

PrepTests ·
PT132.S4.Q18
User Avatar
cwetcholowsky623
Saturday, Oct 14 2023

(P1) Not SEND SPACECRAFTS

(P2) CAN COMMUNICATE -> AS INTELLIGENT

(MC) Therefore, (SENTIENT BEINGS and not AS INTELLIGENT) -> Not DETERMINE

MA: DETERMINE -> (SEND SPACECRAFTS or CAN COMMUNICATE)

(D) formulates this assumption as '(DETERMINE and Not CAN COMMUNICATE) -> SEND SPACECRAFTS'

(B) is a trap and too strong, due to the universal quantification ('any'). What if there are some sentient beings who would want to communicate with us whereas others would not? The former might in fact be willing to communicate with us, even if the latter would not.

PrepTests ·
PT141.S2.Q21
User Avatar
cwetcholowsky623
Friday, Oct 13 2023

(P1) SPANISH 101 -m-> ATTENDED ALL SESSIONS

(P2) Not ATTENDED ALL SESSIONS

MSS:

(IC) SPANISH 101 -m-> ATTENDED ALL SESSIONS -> Not <B-

Answer choice (E) approximates this anticipated conclusion more closely than (C), because 'Not <B-' means 'B- or higher.' Make sure to attentively read all answer choices and to compare them with another!

User Avatar

Saturday, May 13 2023

cwetcholowsky623

PT14.S4.Q25 – Lines in the Peruvian Desert

I personally find this the hardest LR question in PT 14; it is (1) bizarre on the level of content, (2) very long and overloads test takers with information, and (3) at the very end of the fourth section, thus hitting you at a point of the test where you already spent 2+ hours intensively thinking about stuff and are mentally exhausted.

In paraphrased form, the stimulus says:

(1) Phenomenon: In the Peruvian desert, there are different sets of lines in the sand. These lines occur in different layers: On the top layer, there are lines that branch out from a single point. Beneath that, there are lines that form a bird figure.

(2) Hypothesis: An investigator argues for the conclusion that both of these sets of lines were brought about by aliens, who supposedly used the Peruvian desert to land their space ships. To support this conclusion, the investigator evokes the premises that the lines in the sand would have been useless to Incas.

The first thing to do here is to figure out what the stimulus is even about: The phenomenon itself is not immediately clear – it is crucial to note that there are TWO sets of lines, not just one –, and the investigator’s hypothesis is counterintuitive to a degree that it becomes all too easy to disregard the glaring selective attention fallacy in their reasoning (Aliens or Incas, not Incas; therefore aliens). So the first hurdle here is to even figure out what is going on, and to throw out one’s common sense intuitions out of the window (How can you even identify the different layers of ancient lines in the sand? How did the lines stick around for so long? All of these questions become irrelevant).

The next hurdle then is the question stem, which again seems bizarre: Here, the test writers tell us that we seek to establish the conclusion that the lines are supposed to refer to astronomical phenomena, and that we are supposed to block an alternative hypothesis to the effect that the lines are non-astronomical. So at this point this seems to become a sort of strengthen question. The question stem is unusual to an extent that it becomes hard to pre-phrase or anticipate how a right answer might look like. Thus process of elimination seems to be the best approach:

(A) North American natives arranged stones in ways that allow for the measurement of astronomical phenomena. This seems to strengthen a little bit in that it points out a seemingly analogous case (It is not only in South America but also in North America that people used geological means to keep track of astronomical phenomena). However, it seems unclear how this answer choice would also have the blocking effect that the question stem is asking for. Thus keep around as a candidate but expect that one of the other answer choices might well be better.

(B) The straight lines indicate positions at which astronomical events could have been observed ‘at plausible dates,’ and the bird lines could represent a constellation. This gets at both sets of lines and associates both of them with astronomical phenomena. The answer thus is fairly specific. Furthermore, the answer itself postulates its own plausibility (‘plausible dates’), which seems like a massive hint, though again unusual. Like the rest of this question, (B) thus again seems wildly counterintuitive, but in the scenario we are supposed to explain, (B) arguably makes the most sense. In particular, (B) approximates the desired function more than (A). Thus far this thus is the least bad answer choice.

(C) The lines form patterns. This answer choice is worse than (B), due to its lack of specificity and its apparent disconnect from the question stem. Worst answer choice thus far.

(D) Central American Natives used rocks to measure astronomical phenomena. This answer choice seems almost identical to (A) and thus provides good grounds to dismiss both (A) and (D): There can only be one correct answer choice, two virtually identical answer choices thus are likely to both be false.

(E) The bird lines might be older than the straight lines. Again irrelevant; (B) must be right.

Takeaways: This seems to be a question where the LSAT really tries hard to make test takers focus exclusively on reasoning structures, not on common sense intuition or plausibility. In this sense, the question is similar to other early LR questions that seem weird content wise but make syntactical sense on the level of formal logic. Focus on getting a clear understanding of what is going on in the stimulus and the question stem; I spent four minutes on this and still felt overwhelmed. Get a clear grasp of what the phenomenon is, what the explanation attempt from the stimulus is trying to say, and how the two alternative explanatory directions from the question stem relate to another. Then use process of elimination to get through the answers.

User Avatar

Saturday, May 13 2023

cwetcholowsky623

PT14.S4.Q11 – Socialist Observations of History

Conservative: Socialists study history, and they do so to identify trends that inevitably lead to a socialist future. However, this undertaking is certain to fail, because it is only retroactively that historical trends appear inevitable.

Socialist: Socialists do indeed study history, but the purpose of this is practical rather than theoretical: Instead of trying to identify historical trends that themselves bring about socialism, socialists try to identify trends that inform the kind of work that socialists need to do to bring socialism about. Socialism thus is not the inevitable outcome of historical trends, it instead must be worked towards and deliberately brought about.

Under timed conditions this Point at Issue / Disagreement question had me genuinely confused: The conservative and the socialist agree in maintaining that socialists study historical trends, but they disagree about the purpose that these studies are supposed to serve: According to the conservative, these studies are a purely theoretical undertaking, the socialist deems them practical. This thus would have been the issue to anticipate.

The pertinent answer choices are (A) (“[A] socialist society is the inevitable consequence of historical trends that can be identified by an analysis of history”) and (E) (“Socialists analyze history in order to support the view that socialism is inevitable”).

In the case of (E), we do get at a version of the anticipated answer; (E) gets at the conservative’s portraying socialist analyses of history as purely theoretical undertakings, which the socialist rejects.

(A) is more tricky. If (A) said “Socialists believe that a socialist society is the inevitable consequence of historical trends that can be studied,” this arguably would be a right answer choice: The conservative does ascribe this view to socialists, the socialist does not. However, (A) is a claim in itself, not only a belief that socialists may or may not endorse. In this context, the situation is more straightforward: We have no reason to think that the conservative deems the creation of socialist societies inevitable, and the socialist explicitly denies that they are inevitable. So as it stands, the speakers actually seem to agree that (A) is false. This thus can’t be the point at issue.

Takeaways: It is crucial to distinguish clearly between the two viewpoints here, as well as between facts and beliefs. Do not interpret (A) as a belief that the conservative ascribes to socialists; it is rather a claim that the speakers themselves are supposed to endorse or reject.

PrepTests ·
PT140.S1.Q12
User Avatar
cwetcholowsky623
Tuesday, Sep 12 2023

Stimulus:

(P1) WINTER TEMPERATURES ↑ => % RAIN ↑

(MC) Therefore, MELT ↑ => (FLOODS ↑ and WATER↓)

To strengthen this, it would be helpful to have a more explicit connection from the premise to the conclusion, such as (P2) % RAIN ↑ => MELT ↑. (B) approximates an answer along these lines, namely WINTER TEMPERATURES ↑ => (FLOODS ↑ and WATER↓). This answer choice confirms that the presumed original cause indeed leads to the presumed end-effect, even though the anticipated connection between the proportion of rain and melting is not spelled out explicitly.

(A) This answer choice confuses the notions of amount and percentage.

(C) This answer choice does not set out from a region where winter temperatures increased, but rather from a region where they are generally mild.

(D) Out of scope / too general (regions in the world). Also focuses on extreme cases.

(E) Does not seem relevant to the causal reasoning in the argument.

Answer choice ranking: (B) > (C) > (A), (D), (E)

PrepTests ·
PT147.S1.Q10
User Avatar
cwetcholowsky623
Thursday, Oct 12 2023

(P1) SUNLIGHT REFLECT↑ cor. ATMOSPHERE TEMPERATURE ↓

(P2) SURFACE WITH SNOW / ICE ↑ cor. SUNLIGHT REFLECT↑

(MC) Therefore, SURFACE WITH SNOW / ICE ↑ cor. ATMOSPHERE TEMPERATURE ↓

This argument is interesting in that it operates first and foremost with correlations, the causal connections are merely implied. To strengthen it, we can either (1) boost the support that the premises offer for the conclusion, (2) boost the conclusion itself, or (3) boost a premise. Anticipation seems difficult here, but presumably we could e.g. make the causal relations more explicit or just provide independent support for the conclusion.

(A) SNOW FORMATION -> ATMOSPHERE TEMPERATURE ↓. Not sure what this does, I can't see a straightforward impact that this makes on the argument.

(B) There are more factors besides snow and ice that 'affect the cooling' of the atmospheric temperature. Impact again seems unclear, I'm not even sure what 'affecting the cooling' is supposed to mean exactly (Do the factors make the atmosphere even cooler? Do they make the cooling less strong? We don't know).

(C) SURFACE WITH SNOW / ICE ↓ cor. ATMOSPHERE TEMPERATURE ↑. This reaffirms the correlation that we want to establish in the conclusion but identifies a second causal mechanism that brings this correlation about. This seems to strengthen. Not only does the correlation hold because light reflection in ice / snow cools the atmosphere but also because the absence of ice and snow in turn warms the atmosphere. The LSAT seems to be going for path (2) here, we are providing independent support for the conclusion.

(D) The atmosphere gets its heat from light passing through it. Not sure if this strengthens. It might even weaken. If the atmosphere gets its heat from light passing through it, and if snow / ice reflect more light through the atmosphere, wouldn't more snow and ice then heat the atmosphere, rather than cool it? This seems to go in the opposite direction of what we want to establish.

(E) This seems to reiterate premise (P2). Perhaps this strengthens a little bit in that it confirms that this premise really does hold, but for the most part this only seems to repeat information that we already have. By contrast, (C) actually adds something new and boosts the conclusion, rather than just repeating one of the premises.

Ranking: (C) > (E) > (A), (B) > (D)

(C) strengthens the most, (E) perhaps strengthens a little bit, (A) and (B) don't seem to have much impact, (D) arguably weakens.

PrepTests ·
PT126.S1.Q21
User Avatar
cwetcholowsky623
Tuesday, Sep 12 2023

To facilitate filtering through the answer choices here, consider taking a brief note / writing down a conditional as an anticipated answer:

(Not MANY PEOPLE BUY PRODUCTS -> SHOW CANCELED) -> (ANYONE FEELS SHOW IS WORTH PRESERVING -> BUY PRODUCTS)

This takes a little bit of extra time at the start, but without such a pre-phrase the answer choices just get very confusing.

PrepTests ·
PT147.S1.Q23
User Avatar
cwetcholowsky623
Thursday, Oct 12 2023

There are at least two gaps in this argument that we need to bridge to establish the desired conclusion. These gaps relate to the comparability of pre-historic vs. present-day bears (do pre-historic samples accurately represent the animals from which they were taken?) and the comparability of bone vs. blood nitrogen (can we really compare the two to draw conclusions about a given animal kind's feeding habits?). The overall goal is to show that it is in fact legitimate to draw these comparisons, and to thus bolster the conclusion that pre-historic bears' nitrogen levels indicate that they eat meat.

(A) This is about plants, does not address the gaps in the argument or its overall topic.

(B) This is about nitrogen rates (as opposed to levels), and about herbivores. Out of scope, does not address the gaps in the argument.

(C) This is about the numbers of samples taken and thus again misses the gaps in the argument.

(D) Best available answer, establishes that blood and bone samples are comparable.

(E) 'With respect to their nitrogen levels, you can compare the bones of any two meat-eating bears with another.' Unhelpful, we are trying to compare blood nitrogen and bone nitrogen, and we don't know for sure whether the pre-historic bears were in fact carnivores.

Answer choice ranking: (D) > (E) > (A), (B), (C)

User Avatar

Sunday, Mar 12 2023

cwetcholowsky623

PT10.S4.Q24: Rate of Inflation vs. Rate of Return

(1) Most profitable investment: The rate of inflation EXCEEDS the rate of return by a given percentage (say, x%). That is, in real terms, the investment generates a loss; the inflation rate overcompensates whatever profit is being made here. According to the stimulus, this means that the VALUE of this investment declines by the same percentage (x%) at least. Value thus is presented as a function of profit.

(2) Any other investment – that is, any investment that is LESS profitable than the one described in case #1: The value of this investment declines by MORE than x% – that is, the differential between the inflation rate and the rate of return must be even greater than in case #1. Inflation overcompensates the rate of returns even more than in the first case.

Answer choice (C) suggests: The second investment (any investment that is not the most profitable one) is LESS profitable than the most profitable one. If VALUE is a function of PROFIT, and if VALUE in the second case declines more than in the first case, then the second case cannot describe the maximally profitable investment described in case #1.

I’m not sure I’m getting either the economics or the logic behind this right, but it seems to me that a lot of the information presented in the passage is redundant. To conclude what answer choice (C) says ("Case #2 does not describe the most profitable investment"), we only would have needed to know (1) that case #1 is the most profitable investment, and (2) that case #2 can be distinguished from that investment. Is this right / is there a more efficient way to solve this, especially under timed conditions?

User Avatar
cwetcholowsky623
Tuesday, Oct 10 2023

Consider paragraph three: The Third Period is supposed to be characterized by "wild and extreme rhetoric" and the envisioning of a "new world." (B) fits much better than (C) here, due to "final global victory over capitalist oppression."

User Avatar

Friday, Mar 10 2023

cwetcholowsky623

PT9.S2.Q23 - Rich and Poor Farmers

Is the idea that this is a one premise argument where answer choice (A) just contraposes to the desired conclusion? That is, are we to assume that "All rich farmers are dishonest" and "Every honest farmer is poor" are logically equivalent, and that "Every honest farmer is poor" thus serves as a sufficient assumption for "All rich farmers are dishonest?"

PrepTests ·
PT136.S1.P4.Q27
User Avatar
cwetcholowsky623
Monday, Jan 08 2024

(Q27): (A) arguably also is wrong because it is not clear whether the theoretical developments described in the second paragraph took place earlier than the 1930s. Paragraphs one and two do locate these developments in the time period preceding the one from 1934-1939, but these developments still might have taken place during the 1930s, namely from 1930-1934.

So (A) remains ambiguous, not explicitly supported. The passage does not indicate that the physics community in the 1930s neglected "earlier" theoretical developments, because it remains unclear whether the theoretical developments described as neglected occurred prior to 1930.

Also note that these neglected developments themselves were orchestrated by physicists, according to paragraph two. So even if these developments occurred prior to 1930, it would seem questionable whether you could indeed describe them as neglected by the "physics community" as a whole because at least some physicists do in fact seem to endorse them.

By contrast, the correct answer choice (E) is not subject to these ambiguities and thus better supported than (A).

User Avatar

Wednesday, Mar 08 2023

cwetcholowsky623

PT7.S4.Q15 - Ice Cream Consistency vs. Flavor

Why is (E) the right answer choice here, as opposed to (B)?

Stabilizers: Cheap, preserve consistency, but bad for flavor

Low temperatures: Expensive, preserve consistency, better for flavor

(B) suggests: To save money, we should omit the very low temperatures; just use stabilizers to preserve consistency and don't worry about the flavor

Is the idea that the stimulus just remains neutral on flavor, contrary to what (B) suggests?

The stimulus certainly seems to entail the suggestion that cost considerations would encourage ice cream producers to accept higher temperatures, but the stimulus does not also seem to entail an unequivocal recommendation concerning stabilizer use: Contrary to what (B) suggests, ice cream producers might not use very low temperatures (thus reducing costs) AND also not use stabilizers, thus sacrificing consistency for the sake of flavor.

I assume this must be it: The cost considerations mentioned in the stimulus certainly incentivize higher temperatures, but the stimulus does not also entail an unequivocal recommendation concerning stabilizer use. (B) thus does not follow, unlike (E).

PrepTests ·
PT155.S1.Q19
User Avatar
cwetcholowsky623
Thursday, Dec 07 2023

(P1) PRODUCTIVITY GROWTH↓ cor. COMPUTER TECH SPREAD↑

(P2) PRODUCTIVITY GROWTH↓ cor. INDUSTRY RELIES MOST ON COMPUTER TECH

(MC) Therefore, BUSINESS COMPUTER TECH RELIANCE↑ cor. Not (PRODUCTIVITY GROWTH↑)

Weaken

There are a lot of scope shifts in this argument. At first, the author talks about the economies of industrialized countries generally, then they talk about particular industries within these economies, and the conclusion then concerns particular businesses within these industries. This points towards a possible flaw in whole-to-part reasoning: Just because industrialized economies / particular industries generally did not increase their productivity growth, this does not have to mean that the individual businesses within these economies / industries must have failed to do so as well.

The most promising answer choices are (C) and (D).

(C) is about industries that PRODUCE computer technology and increased their productivity growth. This arguably goes after premise (P2) but not the conclusion, since the conclusion talks about industries that INCREASED their reliance on computer tech. It is not clear that the producers of the tech fall within this category. The conclusion also talks about businesses, not about industries, unlike (C).

(D) is about businesses that did in fact increase their reliance on computer tech, and here we learn that within any given industry, BUSINESS COMPUTER TECH RELIANCE↑ cor. PRODUCTIVITY GROWTH↑. This has the same scope as the conclusion and goes in exactly the opposite direction.

(D) thus is better than (C) because (D) in fact talks about the same kind of thing as the conclusion of the argument, and because (D) targets this conclusion itself, rather than a premise. (C) is more open to interpretation and its scope seems woefully different from the conclusion of the argument.

PrepTests ·
PT137.S1.P1.Q7
User Avatar
cwetcholowsky623
Sunday, Jan 07 2024

Preponderance: "Superiority in amount or number; the bulk or majority; also, a large amount or number; an abundance, a profusion."

PrepTests ·
PT137.S4.Q25
User Avatar
cwetcholowsky623
Sunday, Jan 07 2024

Two Flaws:

(1) Too small a sample (Looking at five dentists to make a claim about the dental profession generally).

(2) Scope flaw: Author assumes that "the most effective cavity fighting formula in a toothpaste" automatically qualifies as "the best way to fight cavities" generally. Perhaps there is an even better way to fight cavities that does not involve toothpastes, e.g. flossing or eating a healthy diet?

Only (D) matches both flaws: 10 voters are too small a sample to make a claim about the nation generally; the scope flaw also happens ("Gomez's policies are better than all others, therefore electing Gomez would be the best way to help the nation." Perhaps there are even better ways to help the nation than to vote for a politician who seeks to implement certain policies, e.g. to do volunteer work or to boost the economy?). All other answer choices lack one or both of these flaws.

(A) Small sample size, but the scope flaw does not seem parallel. (A) says: "G would be a popular leader, therefore G would be best for the nation." To be parallel, (A) would have to say: "G would be THE MOST popular leader, therefore G would be best for our nation."

(B) No switch from a small sample to an entire population. A scope flaw does happen ("best policies, therefore best for the nation").

(C) Does not have too small a sample. A scope flaw does happen ("best policies, therefore best for our nation").

(E) Small sample size, but "we" is weird - is "we" here the population as a whole? We don't know; there is no guarantee that this indeed matches the broad generalization in the original argument. "Help the nation" again lacks the comparative element characterizing the scope issue in the original argument. To be parallel, (E) would have to say "Electing G would help the nation more than electing anyone else, therefore electing G is the best course for the nation to follow."

User Avatar

Tuesday, Dec 05 2023

cwetcholowsky623

PT94.S2.Q16 - Heart Disease and Rat Sizes

Based on an examination of three types of rates (small, average size, and large), a recent study found that in rats, SIZE↑ correlates with HEART PROBLEMS↓. In other words, the study found that the greater a rat is, the less likely it is to have heart problems.

RRE EXCEPT. Four of the answer choices must be able to CONTRIBUTE to an explanation of this correlation; one does not. I did not do a pre-phrase here and went straight to the answers.

(A) Compared to large rats, smaller rats are more likely to have fatal diseases that strike earlier than heart problems. Under timed conditions, I took this to suggest: Small rats generally are more likely to die before heart disease strikes, so that heart disease will be overrepresented among the surviving small rats. However, this inference does not follow. If small rats tend to die young, the total NUMBER of surviving small rats that gets heart disease might be smaller, but there is no indication that there would be a corresponding increase in the PROPORTION of small rats that gets heart disease. This answer choice thus does not contribute to an explanation the observed correlation and thus must be right.

(B) Small rats are more likely to have blood vessel issues that causally contribute to heart disease. This helps to explain the correlation.

(C) Larger rats have less stress than smaller ones. If you assume that stress is causally related to heart disease, this contributes to an explanation. Under timed conditions, I thought that this assumption was too big of a jump, but compared to (A) this answer choice still is better. (A) does not contribute to an explanation at all, (C) does so if we make an additional assumption that seems fairly plausible from a common-sense perspective.

(D) The most common cause of heart disease in rats also causes them to be small. This explains the observed correlation by identifying a joint cause of small size and heart disease among rats.

(E) Larger rats do more exercises than smaller rats that causally contribute to heart health. This contributes to an explanation.

(C) is right, (A) is wrong. Under timed conditions, I had taken (A) to lead to a sampling bias making smaller rats not afflicted by heart disease less likely to survive such that heart disease becomes overrepresented among the surviving small rats. However, this inference is false. Just because small rats might be more likely to die for reasons other than heart disease, heart disease does not have to afflict a greater proportion of the surviving rats. I made a mistake here in assessing the implications of this answer choice and then switched to (C) because (C) requires an additional assumption to be explanatory ('Stress causes heart disease').

Takeaways: I originally had chosen the right answer (A) but then switched to (C) after mistakenly making the above-described inference. I likely was overthinking (A). I need to keep an open eye for the distinction between NUMBERS and PROPORTIONs. If unsure, close my eyes for a couple of seconds, do some deep breaths, calm down and reflect. I definitely felt uncomfortable in selecting my answer but could not quite identify what went wrong. NUMBERS vs. PROPORTIONs is a crucial distinction here, similar to e.g. POSSIBILITY vs. ACTUALITY, INATE vs. ACQUIRED, or MENTAL STATE vs. REALITY. Be vigilant, stay alert to these commonly used distinctions.

User Avatar

Tuesday, Dec 05 2023

cwetcholowsky623

PT94.S2.Q2 - Dinosaur Temperatures

(P1) According to dinosaur fossils, dinosaurs had an oxygen isotope ratio in their bones that suggests that their CORES had roughly the same temperature as their LIMBS.

(P2) Today, cold-blooded animals have much warmer CORES than LIMBS.

(MC) Therefore, dinosaurs were probably warm-blooded.

Weaken

This argument assumes, among other things, that warm-blooded animals, unlike cold-blooded animals, do NOT have much warmer CORES than LIMBS, or some other temperature distribution that deviates even more from the dinosaurs'. To anticipate the right answer, I thus was expecting a weaking option targeting this assumption.

(A) Unlike cold-blooded animals, warm-blooded animals only have SLIGHTLY warmer CORES than LIMBS. This goes in the direction of my pre-phrase but is not very strong. Crucially, it remains more likely that dinosaurs were warm-blooded than that they were cold-blooded, just as the author claims. So this answer choice does not seem to actually weaken, even though it gets at the assumption that the author makes, and that I had identified as the weak point of their argument.

(B) Dinosaur fossils don't actually allow you to do the temperature inference described in (P1). This answer is very unusual in that it attacks a premise rather than the reasoning in the argument. Nevertheless, this answer choice definitely weakens, since it takes away the data about dinosaurs that the author presupposes. Keep this answer choice around but be vigilant; see if a less premise-focused answer choice is available.

(C) About oxygen generally. Does not seem to pertain to the argument.

(D) Body temperatures in small and large animals other than dinosaurs. Does not seem to connect directly to the argument; especially since the stimulus does not identify dinosaurs as either small or large.

(E) Warm-blooded animals are more active and use more oxygen than cold-blooded animals. This again does not seem to relate directly to the argument under consideration.

(C), (D), and (E) turn out to be largely unrelated to the argument in the stimulus, and (A) does not seem to weaken the inference made by the author. This leaves (B) as the only remaining answer choice, and thus (B) must be right.

Nevertheless, (B) feels very much uncomfortable and is unusual. (B) just straight up contradicts information that we get in the stimulus, rather than attacking the author's reasoning. It also seems unusual to have this sort of unexpected answer choice so early in the section; just expecting straightforward questions in (Q1)-(Q10) is too naive.

I originally chose (A) because I got too focused on my anticipation of how the right answer could look like, and thus I neglected (B). Nevertheless, a more careful examination of what (A) and (B) are actually saying would have allowed me to get this question right. I need to stay alert to the details of individual answers and compare them against each other; a more thorough examination between (A) and (B) would have allowed me to see that (A) does not in fact weaken and that (B)'s unusual character does not prevent it from being the right answer here. Read answer choices carefully, compare them against each other, and choose the one that has the fewest problems.

User Avatar

Thursday, May 04 2023

cwetcholowsky623

PT12.S4.Q6 - Chronic Fatigue Syndrome

This necessary assumption question discusses the treatment of Chronic Fatigue Syndrome (CFS) with a newly developed drug. CFS is associated with three different symptoms, and we don’t know if these symptoms are the effects of only one virus or of multiple different ones. Tests of the new drug indicate that this drug lessens the severity of all three CFS symptoms. The stimulus takes this to provide evidence to the effect that CFS probably is caused by one single virus, not by multiple different ones.

Pre-phrase / anticipation: We need an assumption to the effect of ‘If a single treatment lessens all of a given syndrome’s symptoms, then it is more likely for this syndrome to be caused by a single virus than by multiple ones.’

The pertinent answer choices are (B) and (D). (B) states: “It is more likely that the new drug counteracts one virus than that it counteracts several viruses.” This matches the consequent in the anticipated assumption but leaves out its antecedent. (B) thus does not make the argument valid and would fall short of being a sufficient assumption. But is (B) necessary? If negated, (B) would indicate that it would be equally likely or even more likely that the new drug affected several viruses. This is not at all what the author is trying to argue and thus would seem to rob their conclusion of any support.

(D) states: “Most syndromes that are characterized by related symptoms are each caused by a single viral infection.” This in itself might be right, and arguably (D) would be a good strengthen answer choice. (D) gets at the conclusion and points out parallel cases where similar correlations have been observed as well. A number of things seem off though: (1) Do we know that the alleviated symptoms in fact are ‘related,’ as this answer choice suggests? We certainly know that they all are effects of one or more causes, but does that also render these effects related to one another? (2) The conclusion in the stimulus takes the results of the experiments with the new drug to provide evidence to the effect that CFS has a single cause, but (D) does not contain a connection to these experiments. Instead, (D) is just making a general claim that arguably strengthens the conclusion in isolation but that does not also connect it to the other parts of the argument.

As an NA answer choice, (B) thus seems better than (D). (B) is essentially saying: In probabilistic terms, the new drug’s acting on three different effects indicates that these three effects likely have a single cause rather than three different ones. By contrast, had the drug only affected two of CFS’s three symptoms, it would have been likely that there are at least two causes for CFS, one virus that triggers two of its symptoms and another virus that triggers the third one. (B) is thus hinting at a sort of appeal to simplicity behind the author's reasoning. The author seems to assume: If two different hypotheses about the causal relationships behind a given correlation are possible, the simpler hypothesis is more likely correct.

PrepTests ·
PT108.S1.P3.Q17
User Avatar
cwetcholowsky623
Monday, Oct 02 2023

In (Q17), there might be a distinction at play between 'The economists' overall conclusion is false' and 'The economists' argument for their conclusion is flawed.' (A) seems to get at the second option, (B) at the first one. The author does not take themselves to have established that it is legitimate to wage moral criticisms at corporations (= falsifying the conclusion), the author merely claims that the economists' rejection of this claim is poorly supported.

Confirm action

Are you sure?